From 3611c8b8510f03c7b87056d1278be4d8db6c0132 Mon Sep 17 00:00:00 2001 From: wieerwill Date: Wed, 2 Mar 2022 16:44:06 +0100 Subject: [PATCH] =?UTF-8?q?Antworten=20f=C3=BCr=20Aufgabe=202?= MIME-Version: 1.0 Content-Type: text/plain; charset=UTF-8 Content-Transfer-Encoding: 8bit --- Kryptographie - Prüfungsvorbereitung.pdf | 4 ++-- Kryptographie - Prüfungsvorbereitung.tex | 5 +++++ 2 files changed, 7 insertions(+), 2 deletions(-) diff --git a/Kryptographie - Prüfungsvorbereitung.pdf b/Kryptographie - Prüfungsvorbereitung.pdf index 8641417..7284bbc 100644 --- a/Kryptographie - Prüfungsvorbereitung.pdf +++ b/Kryptographie - Prüfungsvorbereitung.pdf @@ -1,3 +1,3 @@ version https://git-lfs.github.com/spec/v1 -oid sha256:e6a0b390860665da52e9a86ca7f1f3f30910a685431d26cc639c880626338095 -size 254635 +oid sha256:e6060545051ac802a34323a493c8c2928467c93cfad5b67db45eb093e916f18b +size 261356 diff --git a/Kryptographie - Prüfungsvorbereitung.tex b/Kryptographie - Prüfungsvorbereitung.tex index 0ea6495..71a23a9 100644 --- a/Kryptographie - Prüfungsvorbereitung.tex +++ b/Kryptographie - Prüfungsvorbereitung.tex @@ -178,10 +178,15 @@ \begin{parts} \part Nenne ein informationstheoretisch sicheres Block-Kryptosystem, das von Eva in Szenarium 2 leicht gebrochen werden kann. \begin{solution} + Aus Kenntnis von $x\in\{0,1\}^l$ und $y=e(x,k)$ für ein einziges Paar $(x,k)\in X\times K$ kann Eva den Schlüssel $k=x\oplus_l y$ berechnen. Das gilt für das Cäsar-System, das Vigenère-System und das informationstheoretisch sichere Vernam-System. \end{solution} \part In der Vorlesung wurde possibilistische Sicherheit für Szenarium 2 definiert. Nenne ein $l$-Block-Kryptosystem, das diese Definition erfüllt. Die nötige Schlüsselmenge $K$ hat Größe... \begin{solution} + Ein Kryptosystem $S=(X,K,Y,e,d)$ ist possibilistisch sicher bzgl. Szenarium 2 ,wenn für jedes $1 \leq r\leq |X|$, jede Folge von paarweise verschiedenen Klartexten $x_1,x_2,...,x_r\in X$, jeden Schlüssel $k\in K$ und jedes $y\in Y\backslash\{e(x_i,k)| 1 \leq i < r\}$ ein Schlüssel $k'\in K$ existiert mit $e(x_i,k)=e(x_i,k')$ für alle $1\leq i< r$ und $e(x_r,k')=y$. + + Die nötige Schlüsselmenge $K$ hat Größe $|\{\pi |\pi :X\rightarrow Y\text{ ist injektiv}\}|=\frac{|Y|!}{(|Y|-|X|)!} \geq |X|!$ viele Schlüssel. + Mit $X=\{0,1\}^{128}$ gibt es also $\geq 2^{128}!$ viele Schlüssel. \end{solution} \part Nenne ein Block-Kryptosystem aus der Vorlesung, das gegenwärtig für Szenarium 2 in der Praxis benutzt wird.